679 views
1 votes
1 votes

LB = { < M > | M is a valid Turing Machine } is decidable or undecidable.

2 Answers

0 votes
0 votes
As we can encode $every \,\, Turing\,\, machine$ using $0's$ and $1's$. So weather

 $L_{B} = \left \{ < M > | M\, is\, a\, valid \,Turing\, Machine \right \}$

is $decidable$ or not will depend upon the type of encoding we will use for a Turing machine.
0 votes
0 votes
It is trivial so it is Decidable we cannot use Rice theorem here.

Related questions

0 votes
0 votes
1 answer
1
prasoon054 asked Dec 7, 2023
185 views
Is countable sets part of GATE CS 2024 syllabus?
3 votes
3 votes
2 answers
2
1 votes
1 votes
1 answer
3